How much of a wave function must reside inside event horizon for it to be consumed by the black hole?












3












$begingroup$


Related to this question in astronomy SE (https://astronomy.stackexchange.com/q/30611/10813) and in particular my attempt to answer it, I started to wonder which fraction of a waveform (for eg a photon) need to be inside event horizon in order to make its escape impossible? As far as I understand, there is infinitesimal, yet non-zero part of every photon inside event horizons even right now.










share|cite|improve this question











$endgroup$

















    3












    $begingroup$


    Related to this question in astronomy SE (https://astronomy.stackexchange.com/q/30611/10813) and in particular my attempt to answer it, I started to wonder which fraction of a waveform (for eg a photon) need to be inside event horizon in order to make its escape impossible? As far as I understand, there is infinitesimal, yet non-zero part of every photon inside event horizons even right now.










    share|cite|improve this question











    $endgroup$















      3












      3








      3





      $begingroup$


      Related to this question in astronomy SE (https://astronomy.stackexchange.com/q/30611/10813) and in particular my attempt to answer it, I started to wonder which fraction of a waveform (for eg a photon) need to be inside event horizon in order to make its escape impossible? As far as I understand, there is infinitesimal, yet non-zero part of every photon inside event horizons even right now.










      share|cite|improve this question











      $endgroup$




      Related to this question in astronomy SE (https://astronomy.stackexchange.com/q/30611/10813) and in particular my attempt to answer it, I started to wonder which fraction of a waveform (for eg a photon) need to be inside event horizon in order to make its escape impossible? As far as I understand, there is infinitesimal, yet non-zero part of every photon inside event horizons even right now.







      quantum-mechanics black-holes wavefunction event-horizon






      share|cite|improve this question















      share|cite|improve this question













      share|cite|improve this question




      share|cite|improve this question








      edited 3 hours ago









      Qmechanic

      108k122011253




      108k122011253










      asked 3 hours ago









      tuomastuomas

      1484




      1484






















          2 Answers
          2






          active

          oldest

          votes


















          4












          $begingroup$

          You are thinking of the photon as an extended classical wave, which is not correct within the standard model of particle physics. This is not correct. Photons are point like quantum mechanical particles, and quantum mechanics is all about probabilities.



          A photon approaching a black hole will have a calculable probability to fall through and be captured, or join the photon sphere. This means that a distribution of photons with the same energy and boundary conditions has to be accumulated in order to check the calculation.






          share|cite|improve this answer









          $endgroup$





















            4












            $begingroup$

            Introduction



            First of all we need to establish what the question even means (I will restrict all formulas to non-rotating Schwarzschild black holes of mass $M$ for concreteness). There are no bound states of the electromagnetic field near a black hole, even quasi-stationary states of finite wavelength near the photon radius $r = 3GM/c^2$ will decay, a part falling into the black hole, a part to infinity.



            So the question really is about scattering. It turns out that scattering of electromagnetic waves behaves differently based on wavelength. In SI units the wavelength is $lambda = c/(2pi omega)$ and this is also the localization of the photon. For intuition we can picture the photon as a wavepacket localized in a sphere of radius $lambda$. On the other hand, the typical length of the gravitational background is $GM/c^2$. The main parameter that controls the behavior of the scattering is thus the ratio of these lengths $GM/c^2/(c/(2pi omega))$. I will now switch to geometrized $G=c=1$ units and drop a $2pi$ factor, and then the ratio of the wavelength to the size of the black hole is $sim Momega$.





            Large wavelengths



            For $Momega to 0$ the wavelength is much larger than the black hole, "a very small part of the photon is in the black hole". In this limit it can be computed that the total cross-section of an incoming monochromatic, asymptotically planar wave is
            $$sigma = left[4 pi (2 M)^2 right]frac{4}{3}M^2 omega^2 + mathcal{O}(omega^3)$$
            It is difficult to interpret the leading-order term as some kind of "overlapping volume" of the wave and the black hole, since that would scale as $M^3omega^3$. However, a sphere of radius $R$ will scatter particles with a cross-section $4 pi R^2$. Taking a wave-packet with a centroid that would be classically absorbed by a rigid sphere of radius $2M$, the factor $4 M^2 omega^2/3$ can be loosely understood as the probability it will actually be absorbed. Quite interestingly, the low-energy photon sees the black hole as a two-dimensional surface interacting with its wave-front rather than a three-dimensional volume.





            Shorter wavelengths



            However, this all changes with increasing frequency/decreasing wavelength. The intermediate regime is impossible to compute analytically, so I just put a plot from Crispino et al. (2007) here (I also recommend the paper for further references):
            enter image description here
            In the plot, $sigma_E^{(omega = 0)}$ is the low-frequency limit given above, $sigma_E$ is the total cross-section of monochromatic electromagnetic waves (computed numerically), $sigma_S$ is the same for massless scalar waves, and $sigma_O$ is the optical limit which you get from considering photons as point particles
            $$sigma_O = 27 pi M^2$$
            It is now easy to see that once the size of the wavepacket is comparable or smaller than the black hole, you can simply use the following approximate intuition: If the wavepacket centroid is within the photon sphere, it is going to fall inside it.






            share|cite|improve this answer









            $endgroup$














              Your Answer








              StackExchange.ready(function() {
              var channelOptions = {
              tags: "".split(" "),
              id: "151"
              };
              initTagRenderer("".split(" "), "".split(" "), channelOptions);

              StackExchange.using("externalEditor", function() {
              // Have to fire editor after snippets, if snippets enabled
              if (StackExchange.settings.snippets.snippetsEnabled) {
              StackExchange.using("snippets", function() {
              createEditor();
              });
              }
              else {
              createEditor();
              }
              });

              function createEditor() {
              StackExchange.prepareEditor({
              heartbeatType: 'answer',
              autoActivateHeartbeat: false,
              convertImagesToLinks: false,
              noModals: true,
              showLowRepImageUploadWarning: true,
              reputationToPostImages: null,
              bindNavPrevention: true,
              postfix: "",
              imageUploader: {
              brandingHtml: "Powered by u003ca class="icon-imgur-white" href="https://imgur.com/"u003eu003c/au003e",
              contentPolicyHtml: "User contributions licensed under u003ca href="https://creativecommons.org/licenses/by-sa/3.0/"u003ecc by-sa 3.0 with attribution requiredu003c/au003e u003ca href="https://stackoverflow.com/legal/content-policy"u003e(content policy)u003c/au003e",
              allowUrls: true
              },
              noCode: true, onDemand: true,
              discardSelector: ".discard-answer"
              ,immediatelyShowMarkdownHelp:true
              });


              }
              });














              draft saved

              draft discarded


















              StackExchange.ready(
              function () {
              StackExchange.openid.initPostLogin('.new-post-login', 'https%3a%2f%2fphysics.stackexchange.com%2fquestions%2f475903%2fhow-much-of-a-wave-function-must-reside-inside-event-horizon-for-it-to-be-consum%23new-answer', 'question_page');
              }
              );

              Post as a guest















              Required, but never shown

























              2 Answers
              2






              active

              oldest

              votes








              2 Answers
              2






              active

              oldest

              votes









              active

              oldest

              votes






              active

              oldest

              votes









              4












              $begingroup$

              You are thinking of the photon as an extended classical wave, which is not correct within the standard model of particle physics. This is not correct. Photons are point like quantum mechanical particles, and quantum mechanics is all about probabilities.



              A photon approaching a black hole will have a calculable probability to fall through and be captured, or join the photon sphere. This means that a distribution of photons with the same energy and boundary conditions has to be accumulated in order to check the calculation.






              share|cite|improve this answer









              $endgroup$


















                4












                $begingroup$

                You are thinking of the photon as an extended classical wave, which is not correct within the standard model of particle physics. This is not correct. Photons are point like quantum mechanical particles, and quantum mechanics is all about probabilities.



                A photon approaching a black hole will have a calculable probability to fall through and be captured, or join the photon sphere. This means that a distribution of photons with the same energy and boundary conditions has to be accumulated in order to check the calculation.






                share|cite|improve this answer









                $endgroup$
















                  4












                  4








                  4





                  $begingroup$

                  You are thinking of the photon as an extended classical wave, which is not correct within the standard model of particle physics. This is not correct. Photons are point like quantum mechanical particles, and quantum mechanics is all about probabilities.



                  A photon approaching a black hole will have a calculable probability to fall through and be captured, or join the photon sphere. This means that a distribution of photons with the same energy and boundary conditions has to be accumulated in order to check the calculation.






                  share|cite|improve this answer









                  $endgroup$



                  You are thinking of the photon as an extended classical wave, which is not correct within the standard model of particle physics. This is not correct. Photons are point like quantum mechanical particles, and quantum mechanics is all about probabilities.



                  A photon approaching a black hole will have a calculable probability to fall through and be captured, or join the photon sphere. This means that a distribution of photons with the same energy and boundary conditions has to be accumulated in order to check the calculation.







                  share|cite|improve this answer












                  share|cite|improve this answer



                  share|cite|improve this answer










                  answered 2 hours ago









                  anna vanna v

                  163k8154459




                  163k8154459























                      4












                      $begingroup$

                      Introduction



                      First of all we need to establish what the question even means (I will restrict all formulas to non-rotating Schwarzschild black holes of mass $M$ for concreteness). There are no bound states of the electromagnetic field near a black hole, even quasi-stationary states of finite wavelength near the photon radius $r = 3GM/c^2$ will decay, a part falling into the black hole, a part to infinity.



                      So the question really is about scattering. It turns out that scattering of electromagnetic waves behaves differently based on wavelength. In SI units the wavelength is $lambda = c/(2pi omega)$ and this is also the localization of the photon. For intuition we can picture the photon as a wavepacket localized in a sphere of radius $lambda$. On the other hand, the typical length of the gravitational background is $GM/c^2$. The main parameter that controls the behavior of the scattering is thus the ratio of these lengths $GM/c^2/(c/(2pi omega))$. I will now switch to geometrized $G=c=1$ units and drop a $2pi$ factor, and then the ratio of the wavelength to the size of the black hole is $sim Momega$.





                      Large wavelengths



                      For $Momega to 0$ the wavelength is much larger than the black hole, "a very small part of the photon is in the black hole". In this limit it can be computed that the total cross-section of an incoming monochromatic, asymptotically planar wave is
                      $$sigma = left[4 pi (2 M)^2 right]frac{4}{3}M^2 omega^2 + mathcal{O}(omega^3)$$
                      It is difficult to interpret the leading-order term as some kind of "overlapping volume" of the wave and the black hole, since that would scale as $M^3omega^3$. However, a sphere of radius $R$ will scatter particles with a cross-section $4 pi R^2$. Taking a wave-packet with a centroid that would be classically absorbed by a rigid sphere of radius $2M$, the factor $4 M^2 omega^2/3$ can be loosely understood as the probability it will actually be absorbed. Quite interestingly, the low-energy photon sees the black hole as a two-dimensional surface interacting with its wave-front rather than a three-dimensional volume.





                      Shorter wavelengths



                      However, this all changes with increasing frequency/decreasing wavelength. The intermediate regime is impossible to compute analytically, so I just put a plot from Crispino et al. (2007) here (I also recommend the paper for further references):
                      enter image description here
                      In the plot, $sigma_E^{(omega = 0)}$ is the low-frequency limit given above, $sigma_E$ is the total cross-section of monochromatic electromagnetic waves (computed numerically), $sigma_S$ is the same for massless scalar waves, and $sigma_O$ is the optical limit which you get from considering photons as point particles
                      $$sigma_O = 27 pi M^2$$
                      It is now easy to see that once the size of the wavepacket is comparable or smaller than the black hole, you can simply use the following approximate intuition: If the wavepacket centroid is within the photon sphere, it is going to fall inside it.






                      share|cite|improve this answer









                      $endgroup$


















                        4












                        $begingroup$

                        Introduction



                        First of all we need to establish what the question even means (I will restrict all formulas to non-rotating Schwarzschild black holes of mass $M$ for concreteness). There are no bound states of the electromagnetic field near a black hole, even quasi-stationary states of finite wavelength near the photon radius $r = 3GM/c^2$ will decay, a part falling into the black hole, a part to infinity.



                        So the question really is about scattering. It turns out that scattering of electromagnetic waves behaves differently based on wavelength. In SI units the wavelength is $lambda = c/(2pi omega)$ and this is also the localization of the photon. For intuition we can picture the photon as a wavepacket localized in a sphere of radius $lambda$. On the other hand, the typical length of the gravitational background is $GM/c^2$. The main parameter that controls the behavior of the scattering is thus the ratio of these lengths $GM/c^2/(c/(2pi omega))$. I will now switch to geometrized $G=c=1$ units and drop a $2pi$ factor, and then the ratio of the wavelength to the size of the black hole is $sim Momega$.





                        Large wavelengths



                        For $Momega to 0$ the wavelength is much larger than the black hole, "a very small part of the photon is in the black hole". In this limit it can be computed that the total cross-section of an incoming monochromatic, asymptotically planar wave is
                        $$sigma = left[4 pi (2 M)^2 right]frac{4}{3}M^2 omega^2 + mathcal{O}(omega^3)$$
                        It is difficult to interpret the leading-order term as some kind of "overlapping volume" of the wave and the black hole, since that would scale as $M^3omega^3$. However, a sphere of radius $R$ will scatter particles with a cross-section $4 pi R^2$. Taking a wave-packet with a centroid that would be classically absorbed by a rigid sphere of radius $2M$, the factor $4 M^2 omega^2/3$ can be loosely understood as the probability it will actually be absorbed. Quite interestingly, the low-energy photon sees the black hole as a two-dimensional surface interacting with its wave-front rather than a three-dimensional volume.





                        Shorter wavelengths



                        However, this all changes with increasing frequency/decreasing wavelength. The intermediate regime is impossible to compute analytically, so I just put a plot from Crispino et al. (2007) here (I also recommend the paper for further references):
                        enter image description here
                        In the plot, $sigma_E^{(omega = 0)}$ is the low-frequency limit given above, $sigma_E$ is the total cross-section of monochromatic electromagnetic waves (computed numerically), $sigma_S$ is the same for massless scalar waves, and $sigma_O$ is the optical limit which you get from considering photons as point particles
                        $$sigma_O = 27 pi M^2$$
                        It is now easy to see that once the size of the wavepacket is comparable or smaller than the black hole, you can simply use the following approximate intuition: If the wavepacket centroid is within the photon sphere, it is going to fall inside it.






                        share|cite|improve this answer









                        $endgroup$
















                          4












                          4








                          4





                          $begingroup$

                          Introduction



                          First of all we need to establish what the question even means (I will restrict all formulas to non-rotating Schwarzschild black holes of mass $M$ for concreteness). There are no bound states of the electromagnetic field near a black hole, even quasi-stationary states of finite wavelength near the photon radius $r = 3GM/c^2$ will decay, a part falling into the black hole, a part to infinity.



                          So the question really is about scattering. It turns out that scattering of electromagnetic waves behaves differently based on wavelength. In SI units the wavelength is $lambda = c/(2pi omega)$ and this is also the localization of the photon. For intuition we can picture the photon as a wavepacket localized in a sphere of radius $lambda$. On the other hand, the typical length of the gravitational background is $GM/c^2$. The main parameter that controls the behavior of the scattering is thus the ratio of these lengths $GM/c^2/(c/(2pi omega))$. I will now switch to geometrized $G=c=1$ units and drop a $2pi$ factor, and then the ratio of the wavelength to the size of the black hole is $sim Momega$.





                          Large wavelengths



                          For $Momega to 0$ the wavelength is much larger than the black hole, "a very small part of the photon is in the black hole". In this limit it can be computed that the total cross-section of an incoming monochromatic, asymptotically planar wave is
                          $$sigma = left[4 pi (2 M)^2 right]frac{4}{3}M^2 omega^2 + mathcal{O}(omega^3)$$
                          It is difficult to interpret the leading-order term as some kind of "overlapping volume" of the wave and the black hole, since that would scale as $M^3omega^3$. However, a sphere of radius $R$ will scatter particles with a cross-section $4 pi R^2$. Taking a wave-packet with a centroid that would be classically absorbed by a rigid sphere of radius $2M$, the factor $4 M^2 omega^2/3$ can be loosely understood as the probability it will actually be absorbed. Quite interestingly, the low-energy photon sees the black hole as a two-dimensional surface interacting with its wave-front rather than a three-dimensional volume.





                          Shorter wavelengths



                          However, this all changes with increasing frequency/decreasing wavelength. The intermediate regime is impossible to compute analytically, so I just put a plot from Crispino et al. (2007) here (I also recommend the paper for further references):
                          enter image description here
                          In the plot, $sigma_E^{(omega = 0)}$ is the low-frequency limit given above, $sigma_E$ is the total cross-section of monochromatic electromagnetic waves (computed numerically), $sigma_S$ is the same for massless scalar waves, and $sigma_O$ is the optical limit which you get from considering photons as point particles
                          $$sigma_O = 27 pi M^2$$
                          It is now easy to see that once the size of the wavepacket is comparable or smaller than the black hole, you can simply use the following approximate intuition: If the wavepacket centroid is within the photon sphere, it is going to fall inside it.






                          share|cite|improve this answer









                          $endgroup$



                          Introduction



                          First of all we need to establish what the question even means (I will restrict all formulas to non-rotating Schwarzschild black holes of mass $M$ for concreteness). There are no bound states of the electromagnetic field near a black hole, even quasi-stationary states of finite wavelength near the photon radius $r = 3GM/c^2$ will decay, a part falling into the black hole, a part to infinity.



                          So the question really is about scattering. It turns out that scattering of electromagnetic waves behaves differently based on wavelength. In SI units the wavelength is $lambda = c/(2pi omega)$ and this is also the localization of the photon. For intuition we can picture the photon as a wavepacket localized in a sphere of radius $lambda$. On the other hand, the typical length of the gravitational background is $GM/c^2$. The main parameter that controls the behavior of the scattering is thus the ratio of these lengths $GM/c^2/(c/(2pi omega))$. I will now switch to geometrized $G=c=1$ units and drop a $2pi$ factor, and then the ratio of the wavelength to the size of the black hole is $sim Momega$.





                          Large wavelengths



                          For $Momega to 0$ the wavelength is much larger than the black hole, "a very small part of the photon is in the black hole". In this limit it can be computed that the total cross-section of an incoming monochromatic, asymptotically planar wave is
                          $$sigma = left[4 pi (2 M)^2 right]frac{4}{3}M^2 omega^2 + mathcal{O}(omega^3)$$
                          It is difficult to interpret the leading-order term as some kind of "overlapping volume" of the wave and the black hole, since that would scale as $M^3omega^3$. However, a sphere of radius $R$ will scatter particles with a cross-section $4 pi R^2$. Taking a wave-packet with a centroid that would be classically absorbed by a rigid sphere of radius $2M$, the factor $4 M^2 omega^2/3$ can be loosely understood as the probability it will actually be absorbed. Quite interestingly, the low-energy photon sees the black hole as a two-dimensional surface interacting with its wave-front rather than a three-dimensional volume.





                          Shorter wavelengths



                          However, this all changes with increasing frequency/decreasing wavelength. The intermediate regime is impossible to compute analytically, so I just put a plot from Crispino et al. (2007) here (I also recommend the paper for further references):
                          enter image description here
                          In the plot, $sigma_E^{(omega = 0)}$ is the low-frequency limit given above, $sigma_E$ is the total cross-section of monochromatic electromagnetic waves (computed numerically), $sigma_S$ is the same for massless scalar waves, and $sigma_O$ is the optical limit which you get from considering photons as point particles
                          $$sigma_O = 27 pi M^2$$
                          It is now easy to see that once the size of the wavepacket is comparable or smaller than the black hole, you can simply use the following approximate intuition: If the wavepacket centroid is within the photon sphere, it is going to fall inside it.







                          share|cite|improve this answer












                          share|cite|improve this answer



                          share|cite|improve this answer










                          answered 28 mins ago









                          VoidVoid

                          11.3k11959




                          11.3k11959






























                              draft saved

                              draft discarded




















































                              Thanks for contributing an answer to Physics Stack Exchange!


                              • Please be sure to answer the question. Provide details and share your research!

                              But avoid



                              • Asking for help, clarification, or responding to other answers.

                              • Making statements based on opinion; back them up with references or personal experience.


                              Use MathJax to format equations. MathJax reference.


                              To learn more, see our tips on writing great answers.




                              draft saved


                              draft discarded














                              StackExchange.ready(
                              function () {
                              StackExchange.openid.initPostLogin('.new-post-login', 'https%3a%2f%2fphysics.stackexchange.com%2fquestions%2f475903%2fhow-much-of-a-wave-function-must-reside-inside-event-horizon-for-it-to-be-consum%23new-answer', 'question_page');
                              }
                              );

                              Post as a guest















                              Required, but never shown





















































                              Required, but never shown














                              Required, but never shown












                              Required, but never shown







                              Required, but never shown

































                              Required, but never shown














                              Required, but never shown












                              Required, but never shown







                              Required, but never shown







                              Popular posts from this blog

                              Knooppunt Holsloot

                              Altaar (religie)

                              Gregoriusmis